Stay ahead of learning milestones! Enroll in a class over the summer!

G
Topic
First Poster
Last Poster
Woaah a lot of external tangents
egxa   3
N 26 minutes ago by NO_SQUARES
Source: All Russian 2025 11.7
A quadrilateral \( ABCD \) with no parallel sides is inscribed in a circle \( \Omega \). Circles \( \omega_a, \omega_b, \omega_c, \omega_d \) are inscribed in triangles \( DAB, ABC, BCD, CDA \), respectively. Common external tangents are drawn between \( \omega_a \) and \( \omega_b \), \( \omega_b \) and \( \omega_c \), \( \omega_c \) and \( \omega_d \), and \( \omega_d \) and \( \omega_a \), not containing any sides of quadrilateral \( ABCD \). A quadrilateral whose consecutive sides lie on these four lines is inscribed in a circle \( \Gamma \). Prove that the lines joining the centers of \( \omega_a \) and \( \omega_c \), \( \omega_b \) and \( \omega_d \), and the centers of \( \Omega \) and \( \Gamma \) all intersect at one point.
3 replies
egxa
Apr 18, 2025
NO_SQUARES
26 minutes ago
2023 Hong Kong TST 3 (CHKMO) Problem 4
PikaNiko   3
N 36 minutes ago by lightsynth123
Source: 2023 Hong Kong TST 3 (CHKMO)
Let $ABCD$ be a quadrilateral inscribed in a circle $\Gamma$ such that $AB=BC=CD$. Let $M$ and $N$ be the midpoints of $AD$ and $AB$ respectively. The line $CM$ meets $\Gamma$ again at $E$. Prove that the tangent at $E$ to $\Gamma$, the line $AD$ and the line $CN$ are concurrent.
3 replies
PikaNiko
Dec 3, 2022
lightsynth123
36 minutes ago
9 Did you get into Illinois middle school math Olympiad?
Gavin_Deng   7
N Today at 2:10 AM by anishm2
I am simply curious of who got in.
7 replies
Gavin_Deng
Apr 19, 2025
anishm2
Today at 2:10 AM
Weird Similarity
mithu542   4
N Today at 1:38 AM by EthanNg6
Is it just me or are the 2023 national sprint #21 and 2025 state target #4 strangely similar?
[quote=2023 Natioinal Sprint #21] A right triangle with integer side lengths has perimeter $N$ feet and area $N$ ft^2. What is the arithmetic mean of all possible values of $N$?[/quote]
[quote=2025 State Target #4]Suppose a right triangle has an area of 20 cm^2 and a perimeter of 40 cm. What is
the length of the hypotenuse, in centimeters?[/quote]
4 replies
mithu542
Apr 18, 2025
EthanNg6
Today at 1:38 AM
geometry problem
kjhgyuio   8
N Today at 1:36 AM by EthanNg6
........
8 replies
kjhgyuio
Apr 20, 2025
EthanNg6
Today at 1:36 AM
2025 MATHCOUNTS State Hub
SirAppel   596
N Yesterday at 10:43 PM by Eddie_tiger
Previous Years' "Hubs": (2022) (2023) (2024)Please Read

Now that it's April and we're allowed to discuss ...
[list=disc]
[*] CA: 43 (45 44 43 43 43 42 42 41 41 41)
[*] NJ: 43 (45 44 44 43 39 42 40 40 39 38) *
[*] NY: 42 (43 42 42 42 41 40)
[*] TX: 42 (43 43 43 42 42 40 40 38 38 38)
[*] MA: 41 (45 43 42 41)
[*] WA: 41 (41 45 42 41 41 41 41 41 41 40) *
[*]VA: 40 (41 40 40 40)
[*] FL: 39 (42 41 40 39 38 37 37)
[*] IN: 39 (41 40 40 39 36 35 35 35 34 34)
[*] NC: 39 (42 42 41 39)
[*] IL: 38 (41 40 39 38 38 38)
[*] OR: 38 (44 39 38 38)
[*] PA: 38 (41 40 40 38 38 37 36 36 34 34) *
[*] MD: 37 (43 39 39 37 37 37)
[*] AZ: 36 (40? 39? 39 36)
[*] CT: 36 (44 38 38 36 35 35 34 34 34 33 33 32 32 32 32)
[*] MI: 36 (39 41 41 36 37 37 36 36 36 36) *
[*] MN: 36 (40 36 36 36 35 35 35 34)
[*] CO: 35 (41 37 37 35 35 35 ?? 31 31 30) *
[*] GA: 35 (38 37 36 35 34 34 34 34 34 33)
[*] OH: 35 (41 37 36 35)
[*] AR: 34 (46 45 35 34 33 31 31 31 29 29)
[*] NV: 34 (41 38 ?? 34)
[*] TN: 34 (38 ?? ?? 34)
[*] WI: 34 (40 37 37 34 35 30 28 29 29 29) *
[*] HI: 32 (35 34 32 32)
[*] NH: 31 (42 35 33 31 30)
[*] DE: 30 (34 33 32 30 30 29 28 27 26? 24)
[*] SC: 30 (33 33 31 30)
[*] IA: 29 (33 30 31 29 29 29 29 29 29 29 29 29) *
[*] NE: 28 (34 30 28 28 27 27 26 26 25 25)
[*] SD: 22 (30 29 24 22 22 22 21 21 20 20)
[/list]
Cutoffs Unknown

* means that CDR is official in that state.

Notes

For those asking about the removal of the tiers, I'd like to quote Jason himself:
[quote=peace09]
learn from my mistakes
[/quote]

Help contribute by sharing your state's cutoffs!
596 replies
SirAppel
Apr 1, 2025
Eddie_tiger
Yesterday at 10:43 PM
k NO WAY RICHARD RUSCYK REPLIED TO MY MESSAGE
nmlikesmath   0
Yesterday at 7:50 PM
CHAT THIS IS UNREAL
TYSM RICHARD THANK YOU SO MUCH
0 replies
nmlikesmath
Yesterday at 7:50 PM
0 replies
Website to learn math
hawa   43
N Yesterday at 6:44 PM by anticodon
Hi, I'm kinda curious what website do yall use to learn math, like i dont find any website thats fun to learn math
43 replies
hawa
Apr 9, 2025
anticodon
Yesterday at 6:44 PM
A twist on a classic
happypi31415   10
N Yesterday at 6:22 PM by Maxklark
Rank from smallest to largest: $\sqrt[2]{2}$, $\sqrt[3]{3}$, and $\sqrt[5]{5}$.

Click to reveal hidden text
10 replies
happypi31415
Mar 17, 2025
Maxklark
Yesterday at 6:22 PM
Show that the expression is divisable by 5
Deomad123   5
N Yesterday at 6:20 PM by Maxklark
This was taken from a junior math competition.
$$5|3^{2009} - 7^{2007}$$
5 replies
Deomad123
Mar 25, 2025
Maxklark
Yesterday at 6:20 PM
easy olympiad problem
kjhgyuio   6
N Yesterday at 6:18 PM by Maxklark
Find all positive integer values of \( x \) such that
\[
\sqrt{x - 2011} + \sqrt{2011 - x} + 10
\]is an integer.
6 replies
kjhgyuio
Apr 17, 2025
Maxklark
Yesterday at 6:18 PM
Mathpath acceptance rate
fossasor   15
N Yesterday at 6:15 PM by ZMB038
Does someone have an estimate for the acceptance rate for MathPath?
15 replies
fossasor
Dec 21, 2024
ZMB038
Yesterday at 6:15 PM
Geometric Inequality
sqing   15
N Sep 23, 2015 by sqing
Source: China
In $\triangle ABC$ , prove that\[\frac{1}{h_a}+\frac{1}{h_b}+\frac{1}{h_c}\geq
\frac{2}{\sqrt{3}}(\frac{1}{a}+\frac{1}{b}+\frac{1}{c}).\]
15 replies
sqing
Sep 10, 2015
sqing
Sep 23, 2015
Geometric Inequality
G H J
G H BBookmark kLocked kLocked NReply
Source: China
The post below has been deleted. Click to close.
This post has been deleted. Click here to see post.
sqing
41787 posts
#1 • 1 Y
Y by Adventure10
In $\triangle ABC$ , prove that\[\frac{1}{h_a}+\frac{1}{h_b}+\frac{1}{h_c}\geq
\frac{2}{\sqrt{3}}(\frac{1}{a}+\frac{1}{b}+\frac{1}{c}).\]
Z K Y
The post below has been deleted. Click to close.
This post has been deleted. Click here to see post.
arqady
30210 posts
#2 • 2 Y
Y by Adventure10, Mango247
After using Ravi's substitution we need to prove that
$3(x+y+z)(x+y)^2(x+z)^2(y+z)^2\geq4\left(\sum_{cyc}(x^2+3xy)\right)^2xyz$.
Let $x+y+z=3u$, $xy+xz+yz=3v^2$ and $xyz=w^3$. Hence, we need to prove that $f(w^3)\geq0$,
where $f(w^3)=u(9uv^2-w^3)^2-4(3u^2+v^2)^2w^3$.
We see that $f'(w^3)=-2u(9uv^2-w^3)-4(3u^2+v^2)^2<0$.
Thus, $f$ gets a minimal value for a maximal value of $w^3$, which happens for equality case of two variables.
Let $y=z=1$. Hence, we need to prove $(x-1)^2(x+3)\geq0$. Done!
Z K Y
The post below has been deleted. Click to close.
This post has been deleted. Click here to see post.
xzlbq
15849 posts
#3 • 2 Y
Y by Adventure10, Mango247
Stonger is:

\[ \left( {y}^{2}+10\,yz+{z}^{2} \right)  \left( z+x \right) ^{2}
 \left( x+y \right) ^{2} \left( x+y+z \right) \geq 4\, \left( {x}^{2}+{y}^
{2}+{z}^{2}+3\,xy+3\,yz+3\,xz \right) ^{2}xyz\]
BQ
Z K Y
The post below has been deleted. Click to close.
This post has been deleted. Click here to see post.
xzlbq
15849 posts
#4 • 2 Y
Y by Adventure10, Mango247
\[\left( {z}^{2}+2\,xz+2\,{x}^{2}+2\,xy+{y}^{2} \right) ^{2} \left( x+y
+z \right) ^{2}\geq 2/3\, \left( {z}^{2}+2\,xz+2\,{x}^{2}+{y}^{2}+2\,xy-2
\,yz \right)  \left( {x}^{2}+{y}^{2}+{z}^{2}+3\,xy+3\,yz+3\,xz
 \right) ^{2}\]
=>
In triangle,

\[\frac{1}{k_a} \geq \frac{2}{3}\sqrt{3}(\frac{a}{a+b+c})(\frac{1}{a}+\frac{1}{b}+\frac{1}{c})\]
$k_a=2bc/(b^2+c^2)m_a$
This post has been edited 1 time. Last edited by xzlbq, Sep 11, 2015, 12:22 AM
Z K Y
The post below has been deleted. Click to close.
This post has been deleted. Click here to see post.
xzlbq
15849 posts
#5 • 2 Y
Y by Adventure10, Mango247
\[\frac{1}{w_a}\geq \frac{2\sqrt{3}}{3}\frac{(2h_c+2r_c+h_a+h_b)(h_c+h_a+2h_b+2r_b)(h_a+r_a)a}{\sum{((2h_c+2r_c+h_a+h_b)(h_c+h_a+2h_b+2r_b)(h_a+r_a)a)}}(\frac{1}{a}+\frac{1}{b}+\frac{1}{c})\]
deg=10

BQ
This post has been edited 3 times. Last edited by xzlbq, Sep 11, 2015, 3:09 AM
Z K Y
The post below has been deleted. Click to close.
This post has been deleted. Click here to see post.
szl6208
2032 posts
#6 • 1 Y
Y by Adventure10
xzlbq wrote:
Stonger is:

\[ \left( {y}^{2}+10\,yz+{z}^{2} \right)  \left( z+x \right) ^{2}
 \left( x+y \right) ^{2} \left( x+y+z \right) \geq 4\, \left( {x}^{2}+{y}^
{2}+{z}^{2}+3\,xy+3\,yz+3\,xz \right) ^{2}xyz\]
BQ

We have
\[(y^2+10yz+z^2)(z+x)^2(x+y)^2(x+y+z)-4(x^2+3xy+3xz+y^2+3yz+z^2)^2xyz=\]
\[(x+y+z)(x^2y+x^2z+xy^2-2xyz+xz^2-y^2z-yz^2)^2+4yz(2z+2y+x)(x^2-yz)^2\ge{0}\]
Z K Y
The post below has been deleted. Click to close.
This post has been deleted. Click here to see post.
szl6208
2032 posts
#7 • 1 Y
Y by Adventure10
xzlbq wrote:
\[\left( {z}^{2}+2\,xz+2\,{x}^{2}+2\,xy+{y}^{2} \right) ^{2} \left( x+y
+z \right) ^{2}\geq 2/3\, \left( {z}^{2}+2\,xz+2\,{x}^{2}+{y}^{2}+2\,xy-2
\,yz \right)  \left( {x}^{2}+{y}^{2}+{z}^{2}+3\,xy+3\,yz+3\,xz
 \right) ^{2}\]
=>
In triangle,

\[\frac{1}{k_a} \geq \frac{2}{3}\sqrt{3}(\frac{a}{a+b+c})(\frac{1}{a}+\frac{1}{b}+\frac{1}{c})\]
$k_a=2bc/(b^2+c^2)m_a$

We have
\[(z^2+2xz+2x^2+2xy+y^2)^2(x+y+z)^2-\frac{2}{3}(z^2+2xz+2x^2+y^2+2xy-2yz)(\sum{(x^2+3yz)})^2\]
\[=\frac{1}{3}(x^2+2yz)(y-z)^4+\frac{1}{3}(x^2z+y^3-2y^2z)^2+\frac{1}{3}(x^2y-2yz^2+z^3)^2+\frac{2}{3}x(y+z)(x^2+y^2-3yz+z^2)^2+\frac{1}{3}(8x^2+18xy+18xz+13y^2+20yz+13z^2)(x^2-yz)^2\ge{0}\]
This post has been edited 1 time. Last edited by szl6208, Sep 11, 2015, 5:17 AM
Z K Y
The post below has been deleted. Click to close.
This post has been deleted. Click here to see post.
xzlbq
15849 posts
#8 • 2 Y
Y by Adventure10, Mango247
Stronger is:

\[\frac{1}{\sqrt{h_a}} \geq 
 \frac{\sqrt{2}}{2}(\frac{h_b+h_c}{h_a+h_b+h_c})\sqrt{\sqrt{3}(\frac{1}{a}+\frac{1}{b}+\frac{1}{c})}\]

Click to reveal hidden text

=>
$\left( {\frac {1}{\sqrt {{\it h_a}}}}+{\frac {1}{\sqrt {{\it h_b}}}}+{
\frac {1}{\sqrt {{\it h_c}}}} \right) ^{2}\geq 2\,\sqrt {3} \left( {a}^{-1}
+{b}^{-1}+{c}^{-1} \right) $

BQ
This post has been edited 4 times. Last edited by xzlbq, Sep 11, 2015, 5:29 AM
Z K Y
The post below has been deleted. Click to close.
This post has been deleted. Click here to see post.
szl6208
2032 posts
#9 • 1 Y
Y by Adventure10
xzlbq wrote:
Stronger is:

\[\frac{1}{\sqrt{h_a}} \geq 
 \frac{\sqrt{2}}{2}(\frac{h_b+h_c}{h_a+h_b+h_c})\sqrt{\sqrt{3}(\frac{1}{a}+\frac{1}{b}+\frac{1}{c})}\]

Click to reveal hidden text

=>
$\left( {\frac {1}{\sqrt {{\it h_a}}}}+{\frac {1}{\sqrt {{\it h_b}}}}+{
\frac {1}{\sqrt {{\it h_c}}}} \right) ^{2}\geq 2\,\sqrt {3} \left( {a}^{-1}
+{b}^{-1}+{c}^{-1} \right) $

BQ

We have
\[(z+x)^2(x+y)^2(x^2+y^2+z^2+3xy+3yz+3xz)^2-3(x+y+z)xyz(z+2x+y)^4=\]
\[3x^3(4x^3+y^3+z^3)(y-z)^2+yz(60x^2+7y^2+10yz+7z^2)(x^2-yz)^2+\]
\[\frac{1}{2}(x^2+z^2)(x^3-x^2z+y^3-y^2z)^2+\frac{1}{2}(x^2+y^2)(x^3-x^2y-yz^2+z^3)^2+\]
\[\frac{1}{2}z(18x^5+4x^4y+22x^4z+68x^3z^2+12x^2y^3+64x^2yz^2+44xy^2z^2+12xyz^3+10xz^4+z^5)(x-y)^2+\]
\[\frac{1}{2}y(18x^5+22x^4y+4x^4z+68x^3y^2+64x^2y^2z+12x^2z^3+10xy^4+12xy^3z+44xy^2z^2+y^5)(-z+x)^2+\]
\[\frac{47}{2}x^2z^2(xz-y^2)^2+\frac{47}{2}x^2y^2(xy-z^2)^2\ge{0}\]
Z K Y
The post below has been deleted. Click to close.
This post has been deleted. Click here to see post.
xzlbq
15849 posts
#10 • 1 Y
Y by Adventure10
Szl,Very good!
and kill 12>=deg>=8
BQ
Z K Y
The post below has been deleted. Click to close.
This post has been deleted. Click here to see post.
arqady
30210 posts
#11 • 1 Y
Y by Adventure10
The following inequality is also true.
For all triangle prove that:
$$\frac{1}{l_a}+\frac{1}{l_b}+\frac{1}{l_c}\geq
\frac{2}{\sqrt{3}}\left(\frac{1}{a}+\frac{1}{b}+\frac{1}{c}\right)$$
Z K Y
The post below has been deleted. Click to close.
This post has been deleted. Click here to see post.
xzlbq
15849 posts
#12 • 2 Y
Y by arqady, Adventure10
arqady wrote:
The following inequality is also true.
For all triangle prove that:
$$\frac{1}{l_a}+\frac{1}{l_b}+\frac{1}{l_c}\geq
\frac{2}{\sqrt{3}}\left(\frac{1}{a}+\frac{1}{b}+\frac{1}{c}\right)$$

5#,give a sum,get it.

BQ
Z K Y
The post below has been deleted. Click to close.
This post has been deleted. Click here to see post.
nicusorz
1158 posts
#13 • 2 Y
Y by Adventure10, Mango247
we have :
$ \frac{1}{h_{a}}+\frac{1}{h_{b}}+\frac{1}{h_{ca}}=\frac{1}{r}  $, then
$\frac{1}{r}\geq \frac{2}{\sqrt{3}}\sum \frac{1}{a} $ (Petrovic's inequality )
Z K Y
The post below has been deleted. Click to close.
This post has been deleted. Click here to see post.
sandumarin
1093 posts
#14 • 1 Y
Y by Adventure10
$ LHS=\frac{1}{h_a}+\frac{1}{h_b}+\frac{1}{h_c}=\frac{a}{ah_a}+\frac{b}{bh_b}+\frac{c}{ch_c}=\frac{a+b+c}{2A}=\frac{2s}{2sr}=\frac{1}{r} $ ;where A are triangle area
$ RHS=\frac{2}{\sqrt{3}}(\frac{1}{a}+\frac{1}{b}+\frac{1}{c}=\frac{2}{\sqrt{3}}(\frac{h_a}{ah_a}+\frac{h_b}{bh_b}+\frac{c}{ch_c})=\frac{2}{\sqrt{3}}\frac{h_a+h_b+h_c}{2A}=\frac{h_a+h_b+h_c}{sr\sqrt{3}} $
Ineq returns to:
$ h_a+h_b+h_c\le s\sqrt{3} $ (1)
Because by RAVI's Substitutions result: $ h_a=\frac{2\sqrt{xyz(x+y+z)}}{y+z},h_b=\frac{2\sqrt{xyz(x+y+z)}}{z+x},h_c=\frac{2\sqrt{xyz(x+y+z)}}{x+y} $ where $ x,y,z>0 $
It remains to show that:
$ \frac{2}{y+z}+\frac{2}{z+x}+\frac{2}{x+y}\le\sqrt{3(\frac{1}{yz}+\frac{1}{zx}+\frac{1}{xy})} $
This result using GM ineq and Cauchy-Schwarz:
$ y+z\ge 2\sqrt{yz},z+x\ge 2\sqrt{zx},x+y\ge 2\sqrt{xy}\Rightarrow LHS\le\frac{1}{\sqrt{yz}}+\frac{1}{\sqrt{zx}}+\frac{1}{\sqrt{xy}}\le(by Cauchy-Schwarz)\sqrt{3(\frac{1}{yz}+\frac{1}{zx}+\frac{1}{xy})} $
The proof is ended!
_________
Marin Sandu
Z K Y
The post below has been deleted. Click to close.
This post has been deleted. Click here to see post.
Virgil Nicula
7054 posts
#15 • 2 Y
Y by Adventure10, Mango247
See here (proposed problem) PP6 $:$ the bilateral inequality $\boxed{\ \frac {\sqrt 3}{R}\ \le\ \frac 1a+\frac 1b+\frac 1c\ \le\ \frac {\sqrt 3}{2r}\ }$

what is equivalently with $\boxed{\frac {r\sqrt 3}R\le \frac {h_a+h_b+h_c}{a+b+c}\le \frac {\sqrt 3}2}$ .
This post has been edited 7 times. Last edited by Virgil Nicula, Sep 23, 2015, 2:29 AM
Z K Y
The post below has been deleted. Click to close.
This post has been deleted. Click here to see post.
sqing
41787 posts
#16 • 1 Y
Y by Adventure10
sqing wrote:
In $\triangle ABC$ , prove that\[\frac{1}{h_a}+\frac{1}{h_b}+\frac{1}{h_c}\geq
\frac{2}{\sqrt{3}}(\frac{1}{a}+\frac{1}{b}+\frac{1}{c}).\]
$\sum_{cyc}\frac{1}{h_a}=\frac{1}{abc}\sum_{cyc}\frac{a^2}{sinA}\geq\frac{(\sum_{cyc}a)^2}{abc\sum_{cyc}sinA}\geq\frac{3\sum_{cyc}bc}{abc\sum_{cyc}sinA}\geq\frac{2}{\sqrt{3}}\sum_{cyc}\frac{1}{a}.$
Z K Y
N Quick Reply
G
H
=
a